{ 2 x [4 - (6 ÷ 2)]} x 3

Answers

Answer 1

Answer:

6x^2

Step-by-step explanation:

Answer 2

Answer:

6x²

Step-by-step explanation:

(2x(4 - (6 ÷ 2))) × 3x

(2x(4 - 3)) × 3x

2x(1) × 3x

2x × 3x

6x²


Related Questions

pls help. I will mark BRAINLIEST ​

Answers

The Answer should be Circle , Square


Challenge A bag contains pennies, nickels dimes, and quarters. There are 50 coins in all of the
coins 18% are pennies and 40% are dimes. There are 2 more nickels than pennies. How much
money does the bag contain
The bag contains S

Answers

Answer:

The bag contains $5.14

Step-by-step explanation:

18% of 50 = (.18)(50) = 9 pennies

40% of 50 = (.40)(50) = 20 dimes

2 more nickels than pennies = 9 + 2 = 11 nickels

40 coins so far, so 50 - 40 = 10 quarters

Total value:

9 x ($0.01) = $0.09  for pennies

20 x ($0.10) = $2.00 for dimes

11 x ($0.05) = $0.55 for nickels

10 x ($0.25) = $2.50 for quarters

Grand total value = $5.14


Estimate the payment on a 2019 Honda Civic that is selling for $24,599. You will be leasing the vehicle for 3 years. The estimated resale value of the Honda after 3 years will be $13,750 and the sales tax and fees are estimated at 10%. You will need a 20% down payment and you will be financing the balance for 3 years at 2.8%.

Answers

Answer:

Step-by-step explanation:

Is a triangle with the side lengths 4 feet, 7.5 feet, and 8.5 feet a right triangle? Why or why not?

Answers

Answer: Yes, it is a right triangle

Step-by-step explanation:

Use the pythagorean theorem in order to determine whether this triangle is a right triangle. Plug in the values given in the problem (4, 7.5, and 8.5) into this equation (a^2+b^2=c^2). This gives you 4^2+7.5^2=8.5^2, and this equation is valid because 4^2+7.5^2 does equal 8.5^2.

Answer:

Yes, because 4^2+7.5^2=8.5^2.

Step-by-step explanation:

Slay just did this on my test good luck

This is the second one (I need to get to 80)

Answers

20×36= 720

5×15= 75

75+720= 795

the correct answer is 795 ><

If x = 3 cm and z = 5 cm, what is the length of y? A. 3 cm B. 0.5 cm C. 4 cm D. 5 cm

Answers

5 is your length of y

If x = 3 cm and z = 5 cm, what is the length of y? A. 3 cm B. 0.5 cm C. 4 cm D. 5 cm

ans.C.4cm

What is the best estimate of the sum of fractions? ​

Answers

14 1/2 is what I got

Answer:

13

Step-by-step explanation:

Edgeunity 2021

I'm pretty sure, I just estmated the answer to 13 which was the closest.

Mark borrowed $5,500 at 11.5 percent for five years.

How much did Mark pay in interest?

$1,150

$1,520

$1,757

$2,300

Answers

Answer:

   $ 1 , 150

Step-by-step explanation:

In a popular online role playing game, players can create detailed designs for their
character's "costumes," or appearance. Samuel sets up a website where players can
buy and sell these costumes online. Information about the number of people who
visited the website and the number of costumes purchased in a single day is listed
below.
81 visitors purchased no costume.
17 visitors purchased exactly one costume.
2 visitors purchased more than one costume.
Based on these results, express the probability that the next person will purchase no
more than one costume as a fraction in simplest form.

Answers

Answer: 49/50

Step-by-step explanation:

81 + 17 + 2 = 10

98/100 = 49/50

The probability as a fraction in simplest form is 49/50.

Solve for xxx in the diagram below

Answers

Answer:

x=20°

Step-by-step explanation:

x° + 100° + 3x° = 180° => 4x°+100°=180° => 4x=180°-100°=80° => x=80° : 4 = 20° => x=20°

The value of angle x will be equal to 20°.

What is a supplementary angle?

If two angles sum up to 180 degrees, they are referred to as supplementary angles. A straight angle is created by supplementary angles (180 degrees)

The angle is defined as the span between two intersecting lines or surfaces at or close to the point where they meet.

Given that the three angles are x, 3x, and 100 degrees. The value of the angle x will be calculated as below:-

For supplementary angles, the sum of the angles is 180 degrees.

x° + 100° + 3x° = 180°

4x°+100°=180°

4x=180°-100°=80°

x=80° : 4  

x=20°

Therefore, the value of angle x will be equal to 20°.

To know more about supplementary angles follow

https://brainly.com/question/17706490

#SPJ5

USING THE GRAPHING FUNCTION ON YOUR CALCULATOR WHAT IS THE SOLUTION TO THE SYSTEM OF EQUATIONS SHOWN BELOW? Y+2X=5 4Y=5X=14

Answers

Answer:

x= 2, y= 1

Step-by-step explanation:

took one for the team<3

cuales serían los distintos grupos de personas que podrian sentarse en 24 mesas cuadradas de tal manera que formen otras mesas rectangulares

y en 36 mesas cuadradas​

Answers

Answer:

Step-by-step explanation:

25,000 gallons of my pool needs to be completely drained for maintenance purposes your pump and principal at a rate of 30 gallons a minute let it represent the number of hours the pool has been raining right now equation that can be used to determine the number of hours it will take to complete the drain the pool supposed start raining the pool Monday, May 7 at 8 AM would it be safe to schedule maintenance to begin on Tuesday, May 8 at noon

Answers

Answer:

c

Step-by-step explanation:

Caleb has shelves that hold
a total of 600 boxes. What
percent of the shelves are
Full if he has 60 boxes?

Please help no links thank you!

Answers

Answer:

10%

Step-by-step explanation:

if he has 60 boxes then it would be 60/600

simplify that to 6/60 which further simplifies to 1/10 which is 10%

Answer:

600-100%

60-10%

Odp-Wynosi 10%                                                                                             tep-by-step explanationu  Pozdrawami

HELP WILL MARK BRAINLIEST

Answers

it’s B, you’re welcome:)

Find the missing side of this right
triangle.
х
7
12
x =
Enter the number that belongs in the green box.
Enter

Answers

Answer:

19is the answer I think so please check it iam sorry

x = √144 + 49

x = √193

.........................

F'(x)=2x+17 f(1)=7 what is f(0)

Answers

9514 1404 393

Answer:

  f(0) = -11

Step-by-step explanation:

The function f(x) is found by integrating its derivative. The constant of integration is found from the given point.

  [tex]\displaystyle f(x)=\int{f'(x)}\,dx=\int{(2x+17)}\,dx=x^2+17x+c\\\\f(1)=7=1^2+17\cdot1+c\quad\rightarrow\quad c=-11\\\\f(0)=0^2+17\cdot0-11\\\\\boxed{f(0)=-11}[/tex]

What is the answer to 4 1/3 - 1 1/5

Answers

Answer:

3 2/15 or approximately 3.13

Step-by-step explanation:

4 1/3 - 1 1/5

=13/3 - 6/5

= 47/15

=3 2/15 or approximately 3.13

The first 5 terms in a pattern are
shown below.
25 13 27
-6,-4' 2' 4-7
If the pattern continues, which
equatior. could be used to find the nth
term?

Answers

Answer:

Option B. m(n) = –5.75 –¼n

Step-by-step explanation:

The sequence is given below:

–6, –25/4, –13/2, –27/4, –7

Next, we shall determine the common difference.

Common difference (d) = 2nd – First

2nd term = –25/4

First term = –6

d = –25/4 – – 6

d = –25/4 + 6

d = –6.25 + 6

d = –0.25 = –¼

Finally, we shall determine the nth term

Tₙ = a + (n –1)d

First term (a) = –6

Common difference (d) = –¼

Tₙ = a + (n –1)d

Tₙ = –6 + (n –1)–¼

Tₙ = –6 –¼n + ¼

Tₙ = –6 + ¼ –¼n

Tₙ = –6 + 0.25 –¼n

Tₙ = –5.75 –¼n

Thus, the nth term is given by:

m(n) = –5.75 –¼n

Estimate: 5+81.65
HELP
PLEASE

Answers

Answer: 5+81.65≈85  

How to: Estimate.

Have a great day and stay safe !

John is giving out chocolate to his friends. If he wants to give each friend 23
of a chocolate bar and he has 13 friends, how many chocolate bars will he
need to buy?

Answers

Answer:

299

Step-by-step explanation:

23x13 = 299

Determining the Type of Function
What type of function is represented in the table?

Answers

Answer:

quadratic

Step-by-step explanation:

What is the m angle3 ?

Answers

The measure of m<3 is 105 degrees

Using the law that states that the sum of an interior angle of a rectangle is equal to the exterior.

From the given diagram;

m<X + m<Y = m<3

Given that m<X= 55 degrees and m<Y = 50 degrees

m<3 = 55 + 50

m<3 = 105 degrees

Hence the measure of m<3 is 105 degrees

Learn more here: https://brainly.com/question/24193697

The population of leave town is 1-23,560 and is decreasing at a rate of 1.5%
each
year.
a) when will the population of Leavetown drop below 47,000 Cto the nearest
year)?
b) what will the population of leavetown be years from now?

Answers

Answer: a is correct

Step-by-step explanation:

At 25 years

Step-by-step explanation:

123,000 x 2.375% = 2,921.25.  This gives you the rate that the population is decreasing.  How long will it take to get to 50,000. Do reverse operations to find this answer: 123,000 - 50,000 = 73,000,  the population need to decrease by 73,000 to get to 50,000.  So let's multiply our rate of 2,921.25 x 25 = 73,031  and our final step..  123,000 - 73,031 = 49,969 below 50,000

BRAINLIST PLEASE CAN I GET BRAINLIST

Based on historical data in Oxnard college, we believe that 37% of freshmen do not visit their counselors regularly. For this year, you would like to obtain a new sample to estimate the proportiton of freshmen who do not visit their counselors regularly. You would like to be 98% confident that your estimate is within 3.5% of the true population proportion. How large of a sample size is required

Answers

Answer:

A sample size of 1031 is required.

Step-by-step explanation:

In a sample with a number n of people surveyed with a probability of a success of [tex]\pi[/tex], and a confidence level of [tex]1-\alpha[/tex], we have the following confidence interval of proportions.

[tex]\pi \pm z\sqrt{\frac{\pi(1-\pi)}{n}}[/tex]

In which

z is the zscore that has a pvalue of [tex]1 - \frac{\alpha}{2}[/tex].

The margin of error is of:

[tex]M = z\sqrt{\frac{\pi(1-\pi)}{n}}[/tex]

37% of freshmen do not visit their counselors regularly.

This means that [tex]\pi = 0.37[/tex]

98% confidence level

So [tex]\alpha = 0.02[/tex], z is the value of Z that has a pvalue of [tex]1 - \frac{0.02}{2} = 0.99[/tex], so [tex]Z = 2.327[/tex].

You would like to be 98% confident that your estimate is within 3.5% of the true population proportion. How large of a sample size is required?

A sample size of n is required.

n is found when M = 0.035. So

[tex]M = z\sqrt{\frac{\pi(1-\pi)}{n}}[/tex]

[tex]0.035 = 2.327\sqrt{\frac{0.37*0.63}{n}}[/tex]

[tex]0.035\sqrt{n} = 2.327\sqrt{0.37*0.63}[/tex]

[tex]\sqrt{n} = \frac{2.327\sqrt{0.37*0.63}}{0.035}[/tex]

[tex](\sqrt{n})^2 = (\frac{2.327\sqrt{0.37*0.63}}{0.035})^2[/tex]

[tex]n = 1030.4[/tex]

Rounding up:

A sample size of 1031 is required.

Ed solved a puzzle minute faster than Brett. Brett solved the
puzzle in 98 seconds. How many seconds did it take Ed to solve
the puzzle?
Show your work.​

Answers

Ed took 158 seconds

WORK: Add 60sec. to 98sec.

Mr. Q’s class is having a give away where his students can
receive 100 classroom credits. He will pull names out of a hat and select two winners. Once a student wins, they are not
eligible to win a second time. If there are 40 total people in the class, including you and your
best friend.

What is the probability that both you and your best friend get the 100 classroom credits?

Answers

Answer:

2.5 I think

Step-by-step explanation:

Correct answer will be marked brainliest

find the value of x
4x + 2 = 8​

Answers

Answer:  x = 6/4

Step-by-step explanation:  

4x + 2 = 8

4x = 6

x = 6/4 or 1 1/2

Hope this helped :)

Answer:

x = 1.5, 1 1/2

Step-by-step explanation:

4x + 2 = 8

4x = 6

4x/4 = 6/4

x = 1.5

1.5 = 1 1/2

Have a nice day! :-)

Find the equation of the line. Use exact numbers. y = _x+_

Answers

The answer is y=4x-9

Please help me. I dont have much time! Thank you if you decide to help tho​

Answers

It would either be C or D I think.

Answer:

D. They have the same slope, and the same y-intercept so they have infinitely many solutions.

Step-by-step explanation:

Graphing these lines shows that both lines are the exact same line, and cannot have a point of intersection. Solving the equations shows you a value that is ultimately always correct; such as 0=0. This can be used as proof that "They have the same slope, and the same y-intercept so they have infinitely many solutions" because either equation can be rearranged to equal the other.

Other Questions
Can somebody please help me out please my grads are bad Luigi recorded the temperature in his garden at diffternt times of the same day[tex]please tell \: the \: answer[/tex] How is Abo MOST influenced by preparingmeals in the Islamic Center?What Makes A Meal What are 8 criminal law vocabulary words According to the special theory of relativity, what is the fourth dimension? What did the old woman mean when she told one driver, Im not walking for myself. Im walking for my children and grandchildren. in Claudette Colvin chapter 8 can someone please help me?? as soon as possible! would really appreciate itx = ___ units Select the correct answer.Which expression gives the area of triangle QRS?s1520R9O A 19)(15)cos(209)O B. (11)(15)cos(209)oc. (9)(15)sin(20)OD. (11)(15)sin(209) Lease or Sell Casper Company owns a equipment with a cost of $366,000 and accumulated depreciation of $53,200 that can be sold for $273,400, less a 3% sales commission. Alternatively, Casper Company can lease the equipment to another company for three years for a total of $285,200, at the end of which there is no residual value. In addition, the repair, insurance, and property tax expense that would be incurred by Casper Company on the equipment would total $15,100 over the three years.Prepare a differential analysis on March 23 as to whether Casper Company should lease (Alternative 1) or sell (Alternative 2) the equipment. For those boxes in which you must enter subtracted or negative numbers use a minus sign. Marcus is choosing 4 books to read from a list of 15 books. If order matters, how many ways can he choose to read the books? 4 32,760 23,760 60 Can someone plz help me with this one problem plz plz!!! Question 9 of 10Which best describes how to find the length of an arc in a circle?A. Divide the arc's degree measure by 360, then multiply by thecircumference of the circle,B. Divide the arc's degree measure by 360, then multiply by theradius of the circle.C. Divide the arc's degree measure by 180, then multiply by theradius of the circle.D. Divide the arc's degree measure by 180, then multiply by thecircumference of the circle. PLEASE HURRY!!! ILL GIVE BRAINLIEST!!! 1. Is this a PRIMARY or SECONDARY source?2. What sourcing information proves your answer to #1? Which line segment is a radius of K? Pls whats the answer Define theory and hypothesis? ( Please give a simple answer:) ) Whats the correct answer??? I will mark as brainlest how did kennedy and guevara view the cuban revolution 66 oz = __lb in a mixed number Which of the following taxes are deducted from your paycheck?federalstatelocalall of these